User avatar
 
ohthatpatrick
Thanks Received: 3807
Atticus Finch
Atticus Finch
 
Posts: 4661
Joined: April 01st, 2011
 
This post thanked 7 times.
 
 

Re: Q26 - The government will purchase

by ohthatpatrick Fri Dec 31, 1999 8:00 pm

Let me put a complete, official explanation on this one since there have been a lot of odds and ends questions.

Sufficient Assumption
(task: pick an answer that would PROVE the conclusion)

The conclusion says:
If newspaper is correct, public will be safer.

Okay, well what do we know about either of those ideas?
The newspaper claimed that IF new sirens are installed, the public will be safer.

The conclusion firmly believes that if the newspaper is correct, the public WILL BE safer.

That means that the conclusion firmly believes that the new sirens WILL BE installed.

So to prove the conclusion true, we really have to prove that the new sirens will be installed. *That* is really our goal in this question. How do we prove that the new sirens will be installed?

We have a rule that says:
IF replacement parts hard to obtain --> new sirens installed.

Okay, so we have to prove that replacement parts are hard to obtain.

Anything else useful in the premises?

We have this idea that the company we used to buy parts from has gone out of business.

Does that relate to "parts are hard to obtain"? Yes. So we can expect that the answer is going to try to establish a firm link between:

IF company we used to buy parts from went out of business
THEN Parts are hard to obtain.

IF Parts are hard to obtain
THEN New sirens installed.

IF new sirens installed and newspaper is correct
THEN public will be safer.

We have to prove that new sirens will be installed.

Does (A) do so?
No. It doesn't guarantee we will install new sirens. It just guarantees that IF there were an enhancement in safety, it would be siren-related.

Does (B) do so?
No, it just says that the paper was right about the relationship between new sirens and safety.

Does (C) do so?
Almost. It sounds like parts might be 'hard to obtain' since the old company we used was the only company in the area. But this isn't as explicit as Sufficient Assumption normally would be. How do I know "no companies in the area" = "hard to obtain"? What about Amazon.com?

Does (D) do so?
YES! Explicitly! It establishes that parts are hard to obtain, which guarantees we get new sirens.

Does (E) do so?
Almost. It sounds like the parts might be 'less than optimal' / 'inferior quality' ... but the trigger we need is 'hard to obtain', and this answer choice doesn't give that to us explicitly, as (D) does.

Final answer (D).

Although you could definitely diagram this argument, you can also get through it without diagramming if you keep asking yourself the following questions:
"What idea am I trying to prove?"
"What idea must be established in order to prove that idea?"

In order to prove the public will be safer, I need to establish that new sirens will be installed.

In order to prove that new sirens will be installed, I need to establish that replacement parts are hard to obtain.

The closest piece of info we have for that is "old company went out of business". So I anticipate an answer that tells me "old company went out of business" PROVES that "replacement parts are hard to obtain".


#officialexplanation
 
peg_city
Thanks Received: 3
Forum Guests
 
Posts: 152
Joined: January 31st, 2011
Location: Winnipeg
 
 
trophy
First Responder
 

Q26 - The government will purchase

by peg_city Fri May 20, 2011 3:18 pm

How do I attack this question? I really don't even know where to start.

Thanks
 
theaether
Thanks Received: 23
Jackie Chiles
Jackie Chiles
 
Posts: 44
Joined: January 04th, 2011
 
This post thanked 4 times.
 
trophy
Most Thanked
trophy
First Responder
 

Re: Q26 - The government will purchase

by theaether Fri May 20, 2011 5:31 pm

Looking at the stem, we need a sufficient assumption that will enable the argument to flow from the premises. Let's break down the stimulus.

if replacement is difficult --> gov't purchase new parts
if gov't purchase new parts --> public will be safer

then the one that leaves the largest gap: local company is out of business --> public will be safer.

The stimulus is assuming that because the local company is out of business, that is triggering the logic chain that leads to "public will be safer." But it's not exactly the same thing as either "if replacement is difficult" or "if gov't purchase new parts," either which would legitimately trigger the conclusion "public will be safer."

(D) is the clearest bridge that we're looking for. We know that the local company closed down. And (D) tells us that because of this, we'll have a "difficult to obtain replacement parts" situation that we know for sure guarantees us that "public will be safer." This lets the conclusion flow logically from the premises.

If you think about it like this:
A-->B
B-->C
Therefore, D-->C

Choice D tells us that D-->A. Well this is just what we need then, for D-->A-->B-->C. Another legitimate answer choices can be D-->B, telling us that "Because the local company went out of business, the government will buy modern sirens." There are many many other possibilities for correct answers. You'll see them if you can spot the gap in drawing the conclusion.
 
peg_city
Thanks Received: 3
Forum Guests
 
Posts: 152
Joined: January 31st, 2011
Location: Winnipeg
 
 
trophy
First Responder
 

Re: Q26 - The government will purchase

by peg_city Tue May 24, 2011 3:44 pm

one last question. Why is B wrong?
 
giladedelman
Thanks Received: 833
LSAT Geek
 
Posts: 619
Joined: April 04th, 2010
 
This post thanked 1 time.
 
 

Re: Q26 - The government will purchase

by giladedelman Tue May 24, 2011 11:51 pm

Fantastic explanation!

(B) is wrong because it doesn't add any new information. Notice that the argument says, "if the newspaper is correct, the public will be safer." So it doesn't matter whether the newspaper is correct or not! The argument is only about what will happen if it is.

Another way to think of it is, this answer still leaves a huge gap between "company out of business" and "parts hard to obtain."

Hope that clears it up!
 
levine.jesse
Thanks Received: 0
Vinny Gambini
Vinny Gambini
 
Posts: 3
Joined: June 07th, 2011
 
 
 

Re: Q26 - The government will purchase

by levine.jesse Sat Sep 03, 2011 10:54 pm

Would C be a good answer if it said the only company that sold them rather than the only company in the area that sold them? Is that what's wrong with C?

Thanks, J
 
giladedelman
Thanks Received: 833
LSAT Geek
 
Posts: 619
Joined: April 04th, 2010
 
This post thanked 1 time.
 
 

Re: Q26 - The government will purchase

by giladedelman Wed Sep 07, 2011 5:45 pm

I suppose it would be better, because as it is, (C) leaves open the possibility of getting the parts from someone else. But what if we can actually just get replacement parts for free? So it would still not 100% lock down the idea that parts will be difficult to obtain.
User avatar
 
LSAT-Chang
Thanks Received: 38
Atticus Finch
Atticus Finch
 
Posts: 479
Joined: June 03rd, 2011
 
 
trophy
Most Thankful
trophy
First Responder
 

Re: Q26 - The government will purchase

by LSAT-Chang Sun Sep 18, 2011 6:49 pm

theaether Wrote:if replacement is difficult --> gov't purchase new parts
if gov't purchase new parts --> public will be safer

then the one that leaves the largest gap: local company is out of business --> public will be safer.


I had one very quick question regarding the post made above.
I am always unsure when we can or cannot diagram a core into a conditional or vice versa. Like with the above statement "local company is out of business --> public will be safer", is it a conditional statement?? I thought it wasn't necessarily a conditional statement since there is no linking between the two with a term, but rather they are just two separate sentences. "The local company is out of business" is one fact, and then the author concludes that the "public will be safer." So by linking "local company is out of business" to "if replacement is difficult", aren't we assuming that "local company is out of business --> public will be safer" is a conditional statement? I guess my point is that the conclusion "if the newspaper is correct, the public will be safer" is a conclusion not ONLY derived from "local company is out of business" but also from the conditional statements as diagrammed above (if replacement difficult --> govt purchase new parts --> public safer). So I don't understand how we can link "local company out of business --> public will be safer". Does my point make sense?? The same issue was brought up when I diagrammed it that way for one of the problems, and one of the instructors replied saying I couldn't just link "A --> B" as a conditional because there are other parts that links to B. So B isn't just derived from knowing A, but also C, D, and E (from the stimulus). Any thoughts??
 
jcarloserna
Thanks Received: 0
Forum Guests
 
Posts: 4
Joined: November 12th, 2012
 
 
 

Re: Q26 - The government will purchase

by jcarloserna Fri Nov 16, 2012 4:09 pm

is E incorrect because it overreaches?
 
sumukh09
Thanks Received: 139
Atticus Finch
Atticus Finch
 
Posts: 327
Joined: June 03rd, 2012
 
This post thanked 1 time.
 
trophy
Most Thanked
trophy
First Responder
 

Re: Q26 - The government will purchase

by sumukh09 Fri May 03, 2013 9:34 pm

jcarloserna Wrote:is E incorrect because it overreaches?


E is out of scope - reliability is irrelevant
 
stol1989
Thanks Received: 2
Forum Guests
 
Posts: 20
Joined: October 01st, 2013
 
 
 

Re: Q26 - The government will purchase

by stol1989 Fri Oct 18, 2013 5:19 pm

Maybe my observation can be helpful to someone:

This argument has unusual CONDITIONAL conclusion: "So, if the newspaper is correct, the public will be safer during severe weather in the future".
Notice that argument is not trying to convince us that public will (100%, guarantee) be safer during severe weather in the future. If we misidentify "the public will be safer during severe weather in the future" as conclusion, we are in trouble, because LSAT writers set a trap for us in answer choice "B". "B" triggers "If" (Sufficient) part of conclusion (If the newspaper is correct) and guarantees "then" (necessary) part of conclusion (the public will be safer...).

Answer choice "E" is also trying to use the similar flaw by saying "Hey, the only available parts are of such inferior quality that use of them would make the sirens LESS reliable". Hmm... If sirens would be LESS reliable then public will not be safer! But wait... The government will purchase and install NEW sirens and whole argument is about using NEW sirens so "E" is out.
 
Aquamarine
Thanks Received: 0
Forum Guests
 
Posts: 43
Joined: August 21st, 2013
 
 
 

Re: Q26 - The government will purchase

by Aquamarine Wed Jan 28, 2015 6:38 pm

I'm still confused between C and D. I chose D because I just felt it made more sense, but I still have no idea why D is a better answer than C.

Does the conclusion also make sense if C is an answer just like D?
And if this question is about a necessary assumption instead of a sufficient assumption, can C be an answer?

Please someone enlighten me. Thanks!